0 Daumen
919 Aufrufe

Geben Sie an, ob die Folge monoton ist. Geben Sie im Fall der Monotonie die Art der Monotonie an.

a) \( \left\langle a_{n}\right\rangle=\left\langle\frac{n-1}{n+1}\right\rangle \)
b) \( \left\langle a_{n}\right\rangle=\left\langle n^{2}+2 n-1\right\rangle \)
c) \( \left\langle a_{n}\right\rangle=\left\langle\frac{2 n-1}{n+1}\right\rangle \)
d) \( \left\langle a_{n}\right\rangle=\left\langle\frac{3 n-1}{n^{2}+2}\right\rangle \)
e) \( \left\langle a_{n}\right\rangle=\langle 1,3,5,7, \ldots\rangle \)
f) \( \left\langle a_{n}\right\rangle=\langle 2,3,4, \ldots\rangle \)
g) \( \left\langle a_{n}\right\rangle=\langle 1,-1,1,-1, \ldots\rangle \)
h) \( \left\langle a_{n}\right\rangle=\langle 1,2,2,3,3,3,4,4,4 \ldots\rangle \)
i) \( \left\langle a_{n}\right\rangle=\left\langle 1, \frac{1}{4}, \frac{1}{9}, \frac{1}{16}, \ldots\right\rangle \)
i) \( \quad\left\langle a_{n}\right\rangle=\left\langle\frac{1+3 n}{2 n-1}\right\rangle \)

Stellen Sie die ersten 6 Glieder der Folge grafisch dar (Einheit 1 cm). Welche Art der Monotonie liegt vor? Begründen Sie Ihre Behauptung.

 


Kann ich die obere und untere Grenze hier auch berechnen und wenn wie?

Avatar von

1 Antwort

0 Daumen

z.B. Polynomdivision

an = (n - 1)/(n + 1) = 1 - 2/(n + 1)
(an)' = 2/(n + 1)^2 --> immer Positiv

Streng monoton steigend von a1 = 0 bis lim (n-->∞) an = 1

Wertetabelle für die ersten 6 Glieder kannst du auch alleine oder?

Avatar von 477 k 🚀

Ich habe folgende Glieder berechnet:

Bild Mathematik

Aber ich kann aus diesen Berechnungen keine Obere Grenze feststellen oder?

Kannst du begründen das die Folge streng monoton steigend ist?

Dann weißt du das der Grenzwert für n gegen unendlich die obere grenze ist.

Ich nehme an, dass die Folge desshalb monoton steigend ist weil im Nenner n+1 steht.

Aber rein aus den berechneten Zahlenfolgen kann ich das nicht begründen bzw. nachvollziehen.

Ah ok jetzt verstehe ich:

Durch n+1 ergibt sich die oberste Schranke von 1.

Kannst du die Polynomdivision oder die Ableitung ?

Beides kann helfen die Monotonie zu begründen

 (f/g)´=1 - 2/(n + 1) 

würde ich wohl mit der Quotientenregel ableiten:

R:

 f´*g-f*g´/g²

1*n+1-(1-2)*n/(n+1)²

das müsste die Ableitung sein.

einmal 4 eingesetzt:

1*4+1-(1-2)*4/(4+1)²

5-(-)1*4/25

5+4/25

9/25

0,36

f´(x)>0, somit ist die Folge streng monoton steigend, sofern meine Rechnung stimmt.

Und was nehme ich jetzt als Grenzwert an?


Achtung! Bei meinem Term

an = 1 - 2/(n + 1)

ist die 1 nicht im Bruch. Du hast hier nur 1 - Bruch. Desweiteren machst du leider sehr viele Fehler in der Grundlegenden Klammersetzung.

Wenn ich obigen ableite kann ich es auch schreiben als

an = 1 - 2*(n + 1)^{-1}

Ableiten

(an)' = 0 - 2*(-1)*(n + 1)^{-2}*(1) = 2/(n + 1)^2 > 0 

Wenn die Ableitung (Steigung immer > 0) ist hat man eine streng monotone Funktion. Die Folge ist daher auch streng monoton.

(an)´=-1+1-2*(-1)...

wie komme ich zu der -1 am Ende?

Hast Du nach der Faktorregel abgeleitet oder deshalb die -1 ?

Potenzregel

(x^n)' = n * x^{n - 1}

Man zieht den Exponenten als Faktor vor den Term. Aber was soll deine -1 +1 am Anfang. Das ist doch Unsinn. Das habe ich so auch nicht geschrieben. Man leitet die 1 wegen der Summenregel ab und erhält 0.

$$ a) \left\langle \frac { n-1 }{ n+1 } \right\rangle = \left\langle \frac { n+1 -2 }{ n+1 } \right\rangle = \left\langle 1 - \frac { 2 }{ n+1 } \right\rangle $$
Die Folge ist streng monoton fallend, da der Subtrahend offensichtlich streng monoton steigend ist. (Dies sollte bereits Ergebis der vorherigen Aufgabe sein; eine nochmalige Begründung mithilfe der Differenzialrechnung halte ich daher weder für nötig, noch überhaupt für angemessen!

Die größte untere Schranke ist \(0\) (falls \(n=1\) der Startindex ist),
die kleinste obere Schranke ist \(1\).

Ein anderes Problem?

Stell deine Frage

Willkommen bei der Mathelounge! Stell deine Frage einfach und kostenlos

x
Made by a lovely community